Intersection de deux sous espaces vectoriels

Réponses à toutes vos questions après le Bac (Fac, Prépa, etc.)
sarah79
Membre Relatif
Messages: 187
Enregistré le: 06 Avr 2010, 15:13

intersection de deux sous espaces vectoriels

par sarah79 » 20 Oct 2010, 19:38

Je bloque un peu pour un exercice, voici l'énoncé :

on considère les deux sous espaces vectoriels de R3[x] suivants :
V1=vect(P1,P2,P3) avec P1(X)=X^3+X²+3X+2 , P2(X)=X^3+3X et P3(X)=2X^3+3X²

V2=vect(Q1,Q2,Q3) avec Q1(X)=X^3+X² , Q2(X)=3X^3+X²+2 et Q3(X)=X^3+2X+1

Donner une base de l'intersection V1 inter V2 dans R3[X].

indication : traiter le problème équivalent avec les matrice Mat(Bc)(Pi) et Mat(Bc)(Qj) des polynomes en question dans la base canonique de R3[X], pour se ramener a un énoncé dans R4.

J'ai posé :
mat(Pi)= 2 0 0
3 3 0
1 0 3
1 1 2

et mat(Qj) = 2 0 0
3 3 0
1 0 3
1 1 2

Après je sais pas quoi faire, je dois travaillé avec les colonnes de mes matrices? Comment en trouver une base ?



sarah79
Membre Relatif
Messages: 187
Enregistré le: 06 Avr 2010, 15:13

par sarah79 » 20 Oct 2010, 19:39

Je me suis trompé
mat (Qj)=
0 2 1
0 0 2
1 1 0
1 3 1

arnaud32
Membre Irrationnel
Messages: 1982
Enregistré le: 18 Oct 2010, 16:43

par arnaud32 » 20 Oct 2010, 20:47

bonsoir,

tu dois ecrire tes polynomes graces a leurs coefficients comme des vectuers de R^4.
tu as onc deux matirces 3x4.
tu vas faire une "triangularisation" dessus pour trouver une base "plus simple" avec des polynomes a degres decroissants.

tu verra ainsi dej asi les familles sont libres ou non et donc les reelles dimensions de tes espaces.

Ensuite tu ecris ce que veut dire etre dans V1 et V2 pour chacune des matrices "triangularisee".
et donc ce qu'est etre dans l'intersection.
etc...

sarah79
Membre Relatif
Messages: 187
Enregistré le: 06 Avr 2010, 15:13

par sarah79 » 20 Oct 2010, 20:51

Donc si je comprend il faut travaillé avec les matrices mais tu travailles avec les lignes ou les colonnes des matrices càd tu fait ligne 1 recoit ligne 1 -2ligne 2 (c'est un exemple) ou avec les colonnes? et donc le but est de trouvé une matrice triangulaire c'est ça?

arnaud32
Membre Irrationnel
Messages: 1982
Enregistré le: 18 Oct 2010, 16:43

par arnaud32 » 20 Oct 2010, 21:05

tu fais avec les vecteurs de R^4, donc avec ta notation a toi, avec les colonnes

sarah79
Membre Relatif
Messages: 187
Enregistré le: 06 Avr 2010, 15:13

par sarah79 » 20 Oct 2010, 21:10

Du coup pour la première matrice ça revient a résoudre un système tel que :
2x+3y+z+t=0
3y+t=0
3z+2t=0

oui?

arnaud32
Membre Irrationnel
Messages: 1982
Enregistré le: 18 Oct 2010, 16:43

par arnaud32 » 20 Oct 2010, 21:22

si tu utilises la base canonique de R3[X]
M_1=
1 1 3 2
1 0 3 0
2 3 0 0

M_2 =
1 1 0 0
3 1 0 2
1 0 2 1

si tu fais M_1 x [X^3 , X², X, 1] tu obtiens les trois polynomes P_i

tu vas manipuler les lignes de M_1 et M_2 pour les 'triangulariser', ce qui revient a trouver une base de tes sous espaces ou les polynomes sont de degre decroissant.

sarah79
Membre Relatif
Messages: 187
Enregistré le: 06 Avr 2010, 15:13

par sarah79 » 20 Oct 2010, 21:29

Je comprends ce que vous avez fait mais Mat Bc (Pi) =
2 0 0
3 3 0
1 0 3
1 1 2
d'après ce qui est demander dans l'indication ou je me suis trompé et c'est celle que vous m'avez dit M-1?
Base de R3[x]=1,X,X²,X^3 ?

arnaud32
Membre Irrationnel
Messages: 1982
Enregistré le: 18 Oct 2010, 16:43

par arnaud32 » 20 Oct 2010, 21:32

c'est pareille, ne tinquiete pas c'est juste que j'ecris en ligne, toi en colonnes.

sarah79
Membre Relatif
Messages: 187
Enregistré le: 06 Avr 2010, 15:13

par sarah79 » 20 Oct 2010, 21:34

la matrice ce forme comme:
P1 P2 P3
2 0 0 (1)
3 3 0 (X)
1 0 3 (X²)
1 1 2 (X^3)

sarah79
Membre Relatif
Messages: 187
Enregistré le: 06 Avr 2010, 15:13

par sarah79 » 20 Oct 2010, 21:41

ok travaillé donc avec la matrice ou avec le systeme revient a la meme chose
pour v1 je trouve V1={(x,y,z,t) tel que x-y-2z+3t=0} est ce juste? faut il l'exprimé en fonction d'un polynome?

arnaud32
Membre Irrationnel
Messages: 1982
Enregistré le: 18 Oct 2010, 16:43

par arnaud32 » 20 Oct 2010, 21:59

P1 P2 P3
2 0 0 (1)
3 3 0 (X)
1 0 3 (X²)
1 1 2 (X^3)

que tu peux rearranger en:
0 2 1 (1)
3 0 1 (X)
0 1 0 (X²)
1 0 0 (X^3)

te donne une base de V1 qui prouve par ex que V1 est de dimension 3

pou V2 tu peux trouver:
1 -1 0 (1)
2 -2 1 (X)
0 0 0 (X²)
1 0 0 (X^3)

te donne une base de V2 qui prouve par ex que V2 est de dimension 3


un element de V1 est une combinaison lineaire des 3 vecteurs de la base.
pareil pour V2.
tu prends donc a,b,c pour la combinaison lineaire dans V1; a',b',c' pour celle dans V2
et un element de l'intersection doit verifier l'egalite de ces deux vecteurs...

sarah79
Membre Relatif
Messages: 187
Enregistré le: 06 Avr 2010, 15:13

par sarah79 » 20 Oct 2010, 22:07

Je comprends pas trop comment vous arranger les matrices et pourquoi toutes les deux sont de dimension 3. Le système que j'ai indiqué plus haut est il juste?

sarah79
Membre Relatif
Messages: 187
Enregistré le: 06 Avr 2010, 15:13

par sarah79 » 20 Oct 2010, 22:18

J'ai compris comment on fait avec les matrices mais quand je résoud le système je trouve
z=-2x
t=3x
y=-x
et je trouve que V1=Vect(1,-1,-2,3) dimV1=1 pas 3?? si?

arnaud32
Membre Irrationnel
Messages: 1982
Enregistré le: 18 Oct 2010, 16:43

par arnaud32 » 20 Oct 2010, 22:42

pour la premiere du fais
P2 ; P1-P2 ; -1/6*(P3-3P1+P2)

l'autre fonctionne pareille,
Q3 ; Q1-Q3 ; -1/2*(Q2-Q1-2Q3)


et ensuite tu vas multiplier tes matrices par (a,b,c) et (a',b',c')
et faire les egalite ligne a ligne pour egaliser les polynomes
2b+c a'-b'
3a+c 2a'-2b'+c'
b b'
a c'

tu vas trouver
a=a'
b=b'
c=a-3b
c'=2a-b

tu en conclus donc que c n'est pas independant de a et b.

...

sarah79
Membre Relatif
Messages: 187
Enregistré le: 06 Avr 2010, 15:13

par sarah79 » 20 Oct 2010, 22:57

Je comprend qu'on fait V1 intersection V2 mais je vois pas en quoi ça nous permet de donner une base de V1interV2, maintenant on se retrouve avec 6 paramètre a,b,c,a',b',c'

sarah79
Membre Relatif
Messages: 187
Enregistré le: 06 Avr 2010, 15:13

par sarah79 » 20 Oct 2010, 23:04

La base de V1 inter V2 est (1,0,1) et (0,1,-3)

sarah79
Membre Relatif
Messages: 187
Enregistré le: 06 Avr 2010, 15:13

par sarah79 » 20 Oct 2010, 23:34

quelqu'un peut-il me répondre?

arnaud32
Membre Irrationnel
Messages: 1982
Enregistré le: 18 Oct 2010, 16:43

par arnaud32 » 20 Oct 2010, 23:47

tu en conclus que si tu notes les vecteurs de la base de V1
R1 = P2
R2 = P1-P2
R3 = -1/6*(P3-3P1+P2)

P est dans V1 n V2 ssi P=a*R1+b*R2+c*R3 avec c= a-3b
ssi P= a*R1+b*R2+(a-3b)*R3
ssi P= a*(R1+R3)+b*(R2-3*R3)

ta base est alors (R1+R3) (R2-3*R3) car les vecteurs sont independants

sarah79
Membre Relatif
Messages: 187
Enregistré le: 06 Avr 2010, 15:13

par sarah79 » 20 Oct 2010, 23:56

Je comprends rien à la méthode, je vais revoir ça de mon côté et jreposterais des questions plus tard.

 

Retourner vers ✯✎ Supérieur

Qui est en ligne

Utilisateurs parcourant ce forum : Aucun utilisateur enregistré et 27 invités

Tu pars déja ?



Fais toi aider gratuitement sur Maths-forum !

Créé un compte en 1 minute et pose ta question dans le forum ;-)
Inscription gratuite

Identification

Pas encore inscrit ?

Ou identifiez-vous :

Inscription gratuite